Menu Close

nice-calculus-prove-that-challanging-integral-1-x-1-2-x-dx-ln-2pi-1-x-is-fractional-part-of-x-




Question Number 124723 by mnjuly1970 last updated on 05/Dec/20
               .... nice   calculus....       prove  that::       challanging   integral::      Ω=  ∫_1 ^(  ∞)  ((({x}−(1/2))/x))dx=^(???) ln((√(2π)) )−1     {x} is fractional part of  ′x′
$$\:\:\:\:\:\:\:\:\:\:\:\:\:\:\:….\:{nice}\:\:\:{calculus}…. \\ $$$$\:\:\:\:\:{prove}\:\:{that}:: \\ $$$$\:\:\:\:\:{challanging}\:\:\:{integral}:: \\ $$$$\:\:\:\:\Omega=\:\:\int_{\mathrm{1}} ^{\:\:\infty} \:\left(\frac{\left\{{x}\right\}−\frac{\mathrm{1}}{\mathrm{2}}}{{x}}\right){dx}\overset{???} {=}{ln}\left(\sqrt{\mathrm{2}\pi}\:\right)−\mathrm{1} \\ $$$$\:\:\:\left\{{x}\right\}\:{is}\:{fractional}\:{part}\:{of}\:\:'{x}' \\ $$
Answered by Bird last updated on 05/Dec/20
Ω=∫_1 ^∞ ((x−[x]−(1/2))/x)dx  =∫_1 ^∞ (1−(([x]+(1/2))/x))dx  =Σ_(n=1) ^∞  ∫_n ^(n+1) (1−((n+(1/2))/x))dx  =Σ_(n=1) ^∞  ∫_n ^(n+1) dx−(n+(1/2))(dx/x)  =Σ_(n=1) ^∞ (1−(n+(1/2)){ln(n+1)−ln(n)}  =Σ_(n=1) ^∞ (1−(n+(1/2))ln(1+(1/n)))  ln^′ (1+u)=1−u+o(u^2 ) ⇒  ln(1+u)=u−(u^2 /2) +o(u^2 ) ⇒  ln(1+(1/n))=(1/n)−(1/(2n^2 ))+o((1/n^2 ))⇒  1−(n+(1/2))ln(1+(1/n))  ∼1−(n+(1/2))((1/n)−(1/(2n^2 )))  1−(1−(1/(2n))+(1/(2n))−(1/(4n^2 )))=(1/(4n^2 )) ⇒  Σ_(n=1) ^∞ {1−(n+(1/2))ln(1+(1/n))}  is convergent...be continued
$$\Omega=\int_{\mathrm{1}} ^{\infty} \frac{{x}−\left[{x}\right]−\frac{\mathrm{1}}{\mathrm{2}}}{{x}}{dx} \\ $$$$=\int_{\mathrm{1}} ^{\infty} \left(\mathrm{1}−\frac{\left[{x}\right]+\frac{\mathrm{1}}{\mathrm{2}}}{{x}}\right){dx} \\ $$$$=\sum_{{n}=\mathrm{1}} ^{\infty} \:\int_{{n}} ^{{n}+\mathrm{1}} \left(\mathrm{1}−\frac{{n}+\frac{\mathrm{1}}{\mathrm{2}}}{{x}}\right){dx} \\ $$$$=\sum_{{n}=\mathrm{1}} ^{\infty} \:\int_{{n}} ^{{n}+\mathrm{1}} {dx}−\left({n}+\frac{\mathrm{1}}{\mathrm{2}}\right)\frac{{dx}}{{x}} \\ $$$$=\sum_{{n}=\mathrm{1}} ^{\infty} \left(\mathrm{1}−\left({n}+\frac{\mathrm{1}}{\mathrm{2}}\right)\left\{{ln}\left({n}+\mathrm{1}\right)−{ln}\left({n}\right)\right\}\right. \\ $$$$=\sum_{{n}=\mathrm{1}} ^{\infty} \left(\mathrm{1}−\left({n}+\frac{\mathrm{1}}{\mathrm{2}}\right){ln}\left(\mathrm{1}+\frac{\mathrm{1}}{{n}}\right)\right) \\ $$$${ln}^{'} \left(\mathrm{1}+{u}\right)=\mathrm{1}−{u}+{o}\left({u}^{\mathrm{2}} \right)\:\Rightarrow \\ $$$${ln}\left(\mathrm{1}+{u}\right)={u}−\frac{{u}^{\mathrm{2}} }{\mathrm{2}}\:+{o}\left({u}^{\mathrm{2}} \right)\:\Rightarrow \\ $$$${ln}\left(\mathrm{1}+\frac{\mathrm{1}}{{n}}\right)=\frac{\mathrm{1}}{{n}}−\frac{\mathrm{1}}{\mathrm{2}{n}^{\mathrm{2}} }+{o}\left(\frac{\mathrm{1}}{{n}^{\mathrm{2}} }\right)\Rightarrow \\ $$$$\mathrm{1}−\left({n}+\frac{\mathrm{1}}{\mathrm{2}}\right){ln}\left(\mathrm{1}+\frac{\mathrm{1}}{{n}}\right) \\ $$$$\sim\mathrm{1}−\left({n}+\frac{\mathrm{1}}{\mathrm{2}}\right)\left(\frac{\mathrm{1}}{{n}}−\frac{\mathrm{1}}{\mathrm{2}{n}^{\mathrm{2}} }\right) \\ $$$$\mathrm{1}−\left(\mathrm{1}−\frac{\mathrm{1}}{\mathrm{2}{n}}+\frac{\mathrm{1}}{\mathrm{2}{n}}−\frac{\mathrm{1}}{\mathrm{4}{n}^{\mathrm{2}} }\right)=\frac{\mathrm{1}}{\mathrm{4}{n}^{\mathrm{2}} }\:\Rightarrow \\ $$$$\sum_{{n}=\mathrm{1}} ^{\infty} \left\{\mathrm{1}−\left({n}+\frac{\mathrm{1}}{\mathrm{2}}\right){ln}\left(\mathrm{1}+\frac{\mathrm{1}}{{n}}\right)\right\} \\ $$$${is}\:{convergent}…{be}\:{continued} \\ $$

Leave a Reply

Your email address will not be published. Required fields are marked *